Theorem 2.1.2.2 Higher Topos Theory












3














At the page 74 of HTT, there is the following theorem




Let $S$ be a simplicial set, $mathcal{C}$ a simplicial category, and $phi: mathfrak{C}[S] rightarrow mathcal{C}^{op}$ a simplicial functor. The straightening and unstraigntening functors determine a Quillen adjunction
$$ St_{phi} : (Set_{Delta})_{/S} leftrightarrows Set_{Delta}^{mathcal{C}} :Un_{phi}$$
where $(Set_{Delta})_{/S}$ is endowed with the contravariant model structure and $Set_{Delta}^{mathcal{C}}$ with the projective model structure. [...]




In then says that the proof is easy, but I can't manage to show that $St_{phi}$ sends cofibrations to projective cofibrations. I thought that since the the class of morphisms which are sent to projective cofibrations is weakly saturated it is enough to show the result for all inclusions $partial Delta^n subseteq Delta^n$.



I did not have much success for the simplicial category $mathcal{C}$ and the map $phi$ could be anything and I have a hard time dealing with it.



Furthermore there is something else which troubles me: the model structure on the $Set^{mathcal{C}}_{Delta}$ makes no use of the simplicial enrichement on both $mathcal{C}$ and $sSet$ so I was wondering if I was not missing something by believing that the that model structure on $Set^{mathcal{C}}_{Delta}$ is really the projective model structure coming from the Kan model structure on $sSet$ and not the one coming somehow from an other model stucture using the simplicial enrichement.










share|cite|improve this question




















  • 2




    I agree that it's not easy. Much of chapters 2-4 has been written in easier to read ways since HTT came out. I'd take a look at Moerdijk and Heuts for a more complete argument: arxiv.org/pdf/1308.0704.pdf
    – Kevin Carlson
    Nov 25 '18 at 17:36










  • Thank you for the link, I will take a look at it. However, Lurie seems to have a relatively simple argument in mind and I was really wondering what it could be.
    – Oscar P.
    Nov 26 '18 at 10:25












  • You might find someone to answer this at MO.
    – Kevin Carlson
    Nov 26 '18 at 18:59










  • Cross-posted: mathoverflow.net/questions/316393
    – Watson
    Nov 28 '18 at 10:37
















3














At the page 74 of HTT, there is the following theorem




Let $S$ be a simplicial set, $mathcal{C}$ a simplicial category, and $phi: mathfrak{C}[S] rightarrow mathcal{C}^{op}$ a simplicial functor. The straightening and unstraigntening functors determine a Quillen adjunction
$$ St_{phi} : (Set_{Delta})_{/S} leftrightarrows Set_{Delta}^{mathcal{C}} :Un_{phi}$$
where $(Set_{Delta})_{/S}$ is endowed with the contravariant model structure and $Set_{Delta}^{mathcal{C}}$ with the projective model structure. [...]




In then says that the proof is easy, but I can't manage to show that $St_{phi}$ sends cofibrations to projective cofibrations. I thought that since the the class of morphisms which are sent to projective cofibrations is weakly saturated it is enough to show the result for all inclusions $partial Delta^n subseteq Delta^n$.



I did not have much success for the simplicial category $mathcal{C}$ and the map $phi$ could be anything and I have a hard time dealing with it.



Furthermore there is something else which troubles me: the model structure on the $Set^{mathcal{C}}_{Delta}$ makes no use of the simplicial enrichement on both $mathcal{C}$ and $sSet$ so I was wondering if I was not missing something by believing that the that model structure on $Set^{mathcal{C}}_{Delta}$ is really the projective model structure coming from the Kan model structure on $sSet$ and not the one coming somehow from an other model stucture using the simplicial enrichement.










share|cite|improve this question




















  • 2




    I agree that it's not easy. Much of chapters 2-4 has been written in easier to read ways since HTT came out. I'd take a look at Moerdijk and Heuts for a more complete argument: arxiv.org/pdf/1308.0704.pdf
    – Kevin Carlson
    Nov 25 '18 at 17:36










  • Thank you for the link, I will take a look at it. However, Lurie seems to have a relatively simple argument in mind and I was really wondering what it could be.
    – Oscar P.
    Nov 26 '18 at 10:25












  • You might find someone to answer this at MO.
    – Kevin Carlson
    Nov 26 '18 at 18:59










  • Cross-posted: mathoverflow.net/questions/316393
    – Watson
    Nov 28 '18 at 10:37














3












3








3







At the page 74 of HTT, there is the following theorem




Let $S$ be a simplicial set, $mathcal{C}$ a simplicial category, and $phi: mathfrak{C}[S] rightarrow mathcal{C}^{op}$ a simplicial functor. The straightening and unstraigntening functors determine a Quillen adjunction
$$ St_{phi} : (Set_{Delta})_{/S} leftrightarrows Set_{Delta}^{mathcal{C}} :Un_{phi}$$
where $(Set_{Delta})_{/S}$ is endowed with the contravariant model structure and $Set_{Delta}^{mathcal{C}}$ with the projective model structure. [...]




In then says that the proof is easy, but I can't manage to show that $St_{phi}$ sends cofibrations to projective cofibrations. I thought that since the the class of morphisms which are sent to projective cofibrations is weakly saturated it is enough to show the result for all inclusions $partial Delta^n subseteq Delta^n$.



I did not have much success for the simplicial category $mathcal{C}$ and the map $phi$ could be anything and I have a hard time dealing with it.



Furthermore there is something else which troubles me: the model structure on the $Set^{mathcal{C}}_{Delta}$ makes no use of the simplicial enrichement on both $mathcal{C}$ and $sSet$ so I was wondering if I was not missing something by believing that the that model structure on $Set^{mathcal{C}}_{Delta}$ is really the projective model structure coming from the Kan model structure on $sSet$ and not the one coming somehow from an other model stucture using the simplicial enrichement.










share|cite|improve this question















At the page 74 of HTT, there is the following theorem




Let $S$ be a simplicial set, $mathcal{C}$ a simplicial category, and $phi: mathfrak{C}[S] rightarrow mathcal{C}^{op}$ a simplicial functor. The straightening and unstraigntening functors determine a Quillen adjunction
$$ St_{phi} : (Set_{Delta})_{/S} leftrightarrows Set_{Delta}^{mathcal{C}} :Un_{phi}$$
where $(Set_{Delta})_{/S}$ is endowed with the contravariant model structure and $Set_{Delta}^{mathcal{C}}$ with the projective model structure. [...]




In then says that the proof is easy, but I can't manage to show that $St_{phi}$ sends cofibrations to projective cofibrations. I thought that since the the class of morphisms which are sent to projective cofibrations is weakly saturated it is enough to show the result for all inclusions $partial Delta^n subseteq Delta^n$.



I did not have much success for the simplicial category $mathcal{C}$ and the map $phi$ could be anything and I have a hard time dealing with it.



Furthermore there is something else which troubles me: the model structure on the $Set^{mathcal{C}}_{Delta}$ makes no use of the simplicial enrichement on both $mathcal{C}$ and $sSet$ so I was wondering if I was not missing something by believing that the that model structure on $Set^{mathcal{C}}_{Delta}$ is really the projective model structure coming from the Kan model structure on $sSet$ and not the one coming somehow from an other model stucture using the simplicial enrichement.







category-theory simplicial-stuff higher-category-theory model-categories






share|cite|improve this question















share|cite|improve this question













share|cite|improve this question




share|cite|improve this question








edited Nov 25 '18 at 21:05









Arnaud D.

15.7k52343




15.7k52343










asked Nov 25 '18 at 17:10









Oscar P.

494




494








  • 2




    I agree that it's not easy. Much of chapters 2-4 has been written in easier to read ways since HTT came out. I'd take a look at Moerdijk and Heuts for a more complete argument: arxiv.org/pdf/1308.0704.pdf
    – Kevin Carlson
    Nov 25 '18 at 17:36










  • Thank you for the link, I will take a look at it. However, Lurie seems to have a relatively simple argument in mind and I was really wondering what it could be.
    – Oscar P.
    Nov 26 '18 at 10:25












  • You might find someone to answer this at MO.
    – Kevin Carlson
    Nov 26 '18 at 18:59










  • Cross-posted: mathoverflow.net/questions/316393
    – Watson
    Nov 28 '18 at 10:37














  • 2




    I agree that it's not easy. Much of chapters 2-4 has been written in easier to read ways since HTT came out. I'd take a look at Moerdijk and Heuts for a more complete argument: arxiv.org/pdf/1308.0704.pdf
    – Kevin Carlson
    Nov 25 '18 at 17:36










  • Thank you for the link, I will take a look at it. However, Lurie seems to have a relatively simple argument in mind and I was really wondering what it could be.
    – Oscar P.
    Nov 26 '18 at 10:25












  • You might find someone to answer this at MO.
    – Kevin Carlson
    Nov 26 '18 at 18:59










  • Cross-posted: mathoverflow.net/questions/316393
    – Watson
    Nov 28 '18 at 10:37








2




2




I agree that it's not easy. Much of chapters 2-4 has been written in easier to read ways since HTT came out. I'd take a look at Moerdijk and Heuts for a more complete argument: arxiv.org/pdf/1308.0704.pdf
– Kevin Carlson
Nov 25 '18 at 17:36




I agree that it's not easy. Much of chapters 2-4 has been written in easier to read ways since HTT came out. I'd take a look at Moerdijk and Heuts for a more complete argument: arxiv.org/pdf/1308.0704.pdf
– Kevin Carlson
Nov 25 '18 at 17:36












Thank you for the link, I will take a look at it. However, Lurie seems to have a relatively simple argument in mind and I was really wondering what it could be.
– Oscar P.
Nov 26 '18 at 10:25






Thank you for the link, I will take a look at it. However, Lurie seems to have a relatively simple argument in mind and I was really wondering what it could be.
– Oscar P.
Nov 26 '18 at 10:25














You might find someone to answer this at MO.
– Kevin Carlson
Nov 26 '18 at 18:59




You might find someone to answer this at MO.
– Kevin Carlson
Nov 26 '18 at 18:59












Cross-posted: mathoverflow.net/questions/316393
– Watson
Nov 28 '18 at 10:37




Cross-posted: mathoverflow.net/questions/316393
– Watson
Nov 28 '18 at 10:37















active

oldest

votes











Your Answer





StackExchange.ifUsing("editor", function () {
return StackExchange.using("mathjaxEditing", function () {
StackExchange.MarkdownEditor.creationCallbacks.add(function (editor, postfix) {
StackExchange.mathjaxEditing.prepareWmdForMathJax(editor, postfix, [["$", "$"], ["\\(","\\)"]]);
});
});
}, "mathjax-editing");

StackExchange.ready(function() {
var channelOptions = {
tags: "".split(" "),
id: "69"
};
initTagRenderer("".split(" "), "".split(" "), channelOptions);

StackExchange.using("externalEditor", function() {
// Have to fire editor after snippets, if snippets enabled
if (StackExchange.settings.snippets.snippetsEnabled) {
StackExchange.using("snippets", function() {
createEditor();
});
}
else {
createEditor();
}
});

function createEditor() {
StackExchange.prepareEditor({
heartbeatType: 'answer',
autoActivateHeartbeat: false,
convertImagesToLinks: true,
noModals: true,
showLowRepImageUploadWarning: true,
reputationToPostImages: 10,
bindNavPrevention: true,
postfix: "",
imageUploader: {
brandingHtml: "Powered by u003ca class="icon-imgur-white" href="https://imgur.com/"u003eu003c/au003e",
contentPolicyHtml: "User contributions licensed under u003ca href="https://creativecommons.org/licenses/by-sa/3.0/"u003ecc by-sa 3.0 with attribution requiredu003c/au003e u003ca href="https://stackoverflow.com/legal/content-policy"u003e(content policy)u003c/au003e",
allowUrls: true
},
noCode: true, onDemand: true,
discardSelector: ".discard-answer"
,immediatelyShowMarkdownHelp:true
});


}
});














draft saved

draft discarded


















StackExchange.ready(
function () {
StackExchange.openid.initPostLogin('.new-post-login', 'https%3a%2f%2fmath.stackexchange.com%2fquestions%2f3013094%2ftheorem-2-1-2-2-higher-topos-theory%23new-answer', 'question_page');
}
);

Post as a guest















Required, but never shown






























active

oldest

votes













active

oldest

votes









active

oldest

votes






active

oldest

votes
















draft saved

draft discarded




















































Thanks for contributing an answer to Mathematics Stack Exchange!


  • Please be sure to answer the question. Provide details and share your research!

But avoid



  • Asking for help, clarification, or responding to other answers.

  • Making statements based on opinion; back them up with references or personal experience.


Use MathJax to format equations. MathJax reference.


To learn more, see our tips on writing great answers.





Some of your past answers have not been well-received, and you're in danger of being blocked from answering.


Please pay close attention to the following guidance:


  • Please be sure to answer the question. Provide details and share your research!

But avoid



  • Asking for help, clarification, or responding to other answers.

  • Making statements based on opinion; back them up with references or personal experience.


To learn more, see our tips on writing great answers.




draft saved


draft discarded














StackExchange.ready(
function () {
StackExchange.openid.initPostLogin('.new-post-login', 'https%3a%2f%2fmath.stackexchange.com%2fquestions%2f3013094%2ftheorem-2-1-2-2-higher-topos-theory%23new-answer', 'question_page');
}
);

Post as a guest















Required, but never shown





















































Required, but never shown














Required, but never shown












Required, but never shown







Required, but never shown

































Required, but never shown














Required, but never shown












Required, but never shown







Required, but never shown







Popular posts from this blog

Plaza Victoria

In PowerPoint, is there a keyboard shortcut for bulleted / numbered list?

How to put 3 figures in Latex with 2 figures side by side and 1 below these side by side images but in...